gray's anatomy review: thorax

Réussis tes devoirs et examens dès maintenant avec Quizwiz!

1 A 2-day-old newborn is diagnosed with transposition of the great arteries. Which structure is responsible for the division of the truncus arteriosus into the great arteries? A. Septum secundum B. Septum primum C. Bulbar septum D. Aorticopulmonary septum E. Endocardial cushions

1 D. The aorticopulmonary septum functions to divide the truncus arteriosus and bulbus cordis into the aorta and pulmonary trunk. The septum secundum forms an incomplete separation between the two atria. The septum primum divides the atrium into right and left halves. The bulbar septum is derived from the bulbus cordis and will give rise to the interventricular septum inferior to the aorticopulmonary septum, eventually fusing with it. The endocardial cushions play a role in the division of the AV canal into right and left halves, by causing the AV cushions to approach each other.

10 A 28-year-old woman in her third trimester of pregnancy with a complaint of dizziness for several days is admitted to the hospital. Physical examination reveals that she has diabetes mellitus. Which of the following cardiac malformations is most likely to affect the fetus when the mother has this disease? A. Tetralogy of Fallot B. Transposition of the great arteries C. Atrial septal and ventricular septal defects D. Truncus arteriosus E. Coarctation of the aorta

10 B. Transposition of the great arteries is associated with maternal diabetes. Tetralogy of Fallot and truncus arteriosus are associated with DiGeorge syndrome (22q11). ASDs and VSDs are present in individuals with Down syndrome. Coarctation of the aorta is related to Turner syndrome. Marfan syndrome is present in individuals with aortic atresia.

11 During cardiac catheterization of a 6-year-old child, the radiologist notes that the contrast medium released into the arch of the aorta is visible immediately in the left pulmonary artery. What is the most likely explanation for this finding? A. Atrial septal defect B. Mitral stenosis C. Patent ductus arteriosus D. Patent ductus venosus E. Ventricular septal defect

11 C. The ductus arteriosus is an embryologic structure that acts as a communication between the pulmonary trunk and the aorta. If it remains patent, the injected contrast medium would flow from the aorta through this communication and into the pulmonary artery. An atrial septal defect is a communication between the atria. Mitral stenosis is a narrowing of the AV valve between the left atrium and left ventricle. The ductus venosus transports blood from the left umbilical vein to the inferior vena cava, bypassing the liver. Ventricular septal defect is a communication between the ventricles.

12 A 3-year-old male patient presents with a clinically significant atrial septal defect (ASD). The ASD usually results from incomplete closure of which of the following structures? A. Foramen ovale B. Ligamentum arteriosum C. Ductus arteriosus D. Sinus venarum E. Coronary sinus

12 A. An atrial septal defect (ASD) is a communication between the right and left atria. In the formation of the partition between the two atria, the opening in the foramen secundum, also known as the foramen ovale, typically closes at birth. If it remains patent, an ASD will result. The rest of the structures are not associated with atrial septal defects.

13 A premature infant has progressive difficulty in breathing and is diagnosed with respiratory distress syndrome. Which cells are deficient in synthesizing surfactant in this syndrome? A. Alveolar capillary endothelial B. Bronchial mucous C. Bronchial respiratory epithelium D. Type I alveolar E. Type II alveolar

13 E. Type II alveolar cells are the only cells that produce surfactant.

14 A newborn baby was diagnosed with eventration of the diaphragm. In this condition, half of the diaphragm ascends into the thorax during inspiration, while the other half contracts normally. What is the cause of this condition? A. Absence of a pleuropericardial fold B. Absence of musculature in one half of the diaphragm C. Failure of migration of diaphragm D. Failure of the septum transversum to develop E. Absence of a pleuroperitoneal fold

14 B. An absence of musculature in one half of the diaphragm causes it to protrude into the thoracic cavity forming a pouch into which the abdominal viscera protrude. Pleuropericardial folds are responsible for separating the pericardial cavity from the pleural cavity. Typically, the diaphragm migrates to its position with the fibrous pericardium. The septum transversum is the primordial central tendon of the diaphragm that separates the heart from the liver. The pleuroperitoneal folds form the pleuroperitoneal membranes that separate the pleural cavity from the peritoneal cavity. Absence of any of these would not have anything to do with eventration of theiaphragm.

15 A 35-year-old male is admit ed to the emergency department because of a severe nosebleed and a headache that had become worse during the weekend. On physical examination his upper body appears much bet er developed than his lower body, a loud midsystolic murmur is present on his anterior chest wall and back, his lower extremities are cold, and femoral pulses are absent. Which of the fol owing embryologic structure(s) has been most likely af ected to produce such symptoms? A. Bulbus cordis B. Ductus arteriosus C. Third, fourth, and sixth pharyngeal arches D. Right and left horns of sinus venosus E. Right cardinal vein

15 C. The anomalies present in this individual are all caused by a coarctation of the aorta. The portion of the aortic arch that is constricted arises from the third, fourth, and sixth pharyngeal arches. The bulbus cordis becomes part of the ventricular system. The ductus arteriosus becomes the ligamentum arteriosum.

16 After a 2-day-old newborn male swallows milk he becomes cyanotic. After 3 days he develops pneumonia. A tracheoesophageal fistula is suspected. Failure of development has occurred most specifically in which of the following structures? A. Esophagus B. Trachea C. Tongue D. Tracheoesophageal septum E. Pharynx

16 D. The tracheoesophageal septum is a primordial structure that separates the trachea from the esophagus. If this structure fails to develop, a tracheoesophageal fistula will result, in which event the two structures will not separate completely. When the infant attempts to swallow milk, it spills into the esophageal pouch and is regurgitated. The child becomes cyanotic because an insufficient amount of oxygen is reaching the lungs as a result of the malformed trachea.

17 After a 2-day-old newborn male swallows milk he becomes cyanotic. After 3 days he develops pneumonia. A tracheoesophageal fistula is suspected. Which of the following conditions is most likely to be associated with a tracheoesophageal fistula? A. Oligohydramnios B. Rubella C. Polyhydramnios D. Thalidomide E. Toxoplasmosis

17 C. Polyhydramnios is an excess of amniotic fluid, often associated with esophageal atresia or a tracheoesophageal fistula. This abnormality affects fetal ability to swallow the normal amount of amniotic fluid; therefore, excess fluid remains in the amniotic sac. None of the other factors listed has an association with this type of fistula.

18 A 2-day-old newborn male develops mild cyanosis. An ultrasound examination reveals a patent ductus arteriosus. Which of the following infections will most likely lead to this congenital anomaly? A. Toxoplasmosis B. Rubella C. Cytomegalovirus D. Varicella virus E. Treponema pallidum

18 B. Congenital heart defects are common problems that can be caused by teratogens, such as the rubella virus, or single-gene factors or chromosomal abnormalities.

19 A 5-year-old boy has frequent episodes of fatigability and dyspnea. An ultrasound examination reveals an atrial septal defect, located at the opening of the superior vena cava. Which of the following types of atrial septal defects are characteristic for this description? A. Ostium secundum B. Ostium primum C. Atrioventricular (AV) canal D. Common atrium E. Sinus venosus

19 E. Sinus venosus atrial septal defects occur close to the entry of the superior vena cava in the superior portion of the interatrial septum. Ostium secundum atrial septal defects are located near the fossa ovale and encompass both septum primum and septum secundum defects. An ostium primum defect is a less common form of atrial septal defect and is associated with endocardial cushion defects because the septum primum fails to fuse with the endocardial cushions, resulting in a patent foramen primum. An AV canal defect is not a clinically significant type of atrial septal defect. A common atrium is an uncommon type of atrial septal defect in which the interatrial septum is absent.

2 A 32-year-old woman in her third trimester of pregnancy is undergoing a routine ultrasound examination. The examination of the fetus reveals enlarged and echogenic lungs, inverted diaphragm, and fetal ascites. Which condition is best characterized by these signs? A. Laryngeal atresia B. Tracheal atresia C. Polyhydramnios D. Lung hypoplasia E. Oligohydramnios

2 A. Laryngeal atresia (congenital high airway obstruction syndrome) is a rare obstruction of the upper fetal airway. Distal to the site of the atresia, the airways dilate, lungs enlarge and become echogenic, the diaphragm flattens or inverts, and fetal ascites and/or hydrops develop. Tracheal atresia is a rare obstruction of the trachea, commonly found with a tracheoesophageal fistula, probably resulting from the unequal division of foregut into esophagus and trachea. Polyhydramnios is an excess of amniotic fluid, often associated with esophageal atresia or tracheoesophageal fistula. Lung hypoplasia is reduced lung volume, often seen in infants with a congenital diaphragmatic hernia. Oligohydramnios, or a decrease in amniotic fluid, is associated with stunted lung development and pulmonary hypoplasia.

20 A 3-day-old newborn was born with ectopia cordis. Despite the efforts of doctors at the pediatric intensive care unit the infant died from cardiac failure and hypoxemia. Which of the following embryologic events is most likely responsible for the development of such conditions? A. Faulty development of the sternum and pericardium, secondary to incomplete fusion of the lateral folds B. Interruption of third pharyngeal arch development C. Interruption of fourth pharyngeal arch development D. Interruption of fifth pharyngeal arch development E. Faulty development of sinus venosus

20 A. Ectopia cordis is a condition in which the heart is located abnormally outside the thoracic cavity, commonly resulting from a failure of fusion of the lateral folds in forming the thoracic wall. This is incompatible with life because of the occurrence of infection, cardiac failure, or hypoxemia. Faulty development of the sinus venosus is related to atrial septal defects that result from deficient absorption of the sinus venosus into the right atrium and/or unusual development of the septum secundum.

21 A 2-day-old newborn male is admitted to the pediatric intensive care unit with cyanosis and tachypnea. Cardiac ultrasound and MRI examinations reveal totally anomalous pulmonary connections. Which of the following embryologic events is responsible for this malformation? A. Abnormal septation of the sinus venosus B. Abnormal development of the septum secundum C. Abnormal development of the left sinus horn D. Abnormal development of the coronary sinus E. Abnormal development of common cardinal vein

21 A. The right horn of the sinus venosus has two divisions: One develops into the sinus venarum, the smooth interior aspect of the right atrial wall; the other half develops into the pulmonary veins. Abnormal septation of the sinus venosus can lead to inappropriate pulmonary connections. Abnormal development of the left sinus horn would present with abnormalities in the coronary sinus, whereas incorrect development of the septum secundum can result in an atrial septal defect but would not be involved with anomalous pulmonary veins. The left sinual horn develops into the coronary sinus, and the right sinual horn is incorporated into the right atrial wall.

22 A 3-day-old newborn has difficulties breathing. A CT scan of his chest and abdomen reveals the absence of the central tendon of the diaphragm. Which of the following structures failed to develop normally? A. Pleuroperitoneal folds B. Pleuropericardial folds C. Septum transversum D. Cervical myotomes E. Dorsal mesentery of the esophagus

22 C. The septum transversum is a thickened layer of mesoderm that gives origin to the central tendon of the diaphragm. It is situated between the thoracic cavity and the omphaloenteric duct. As the lungs grow into the pericardio-peritoneal canal, they give rise to two folds: the pleuroperitoneal and pleuropericardial folds. The pleuroperitoneal folds are responsible for formation of the posterolateral aspect of the diaphragm, and the pleuropericardial folds develop into the fibrous pericardium. The crura provide origin of the dorsal mesentery of the esophagus, whereas the cervicalyotomes are responsible for the musculature of the diaphragm. (Note that these are cervical myotomes C3 to C5, the levels of origin of the phrenic nerve.)

23 A 30-year-old man is diagnosed with a blockage of arterial flow in the proximal part of the thoracic aorta. Brachial arterial pressure is markedly increased, femoral pressure is decreased, and the femoral pulses are delayed. The patient shows no external signs of inflammation. Which of the following structures failed to develop normally? A. Second aortic arch B. Third aortic arch C. Fourth aortic arch D. Fifth aortic arch E. Ductus venosus

23 C. The fourth aortic arch develops into the aortic arch on the left side and the brachiocephalic and subclavian arteries on the right side of the embryo. Improper development of the arch of the aorta will cause an increased pressure in the subclavian artery and, subsequently, the brachial artery. Similarly, decreased flow through the aorta will lead to a decreased pressure in the femoral artery. The second aortic arch, specifically the dorsal aspect, develops into aspects of the small stapedial artery. The proximal part of the third aortic arch gives rise to the common carotid arteries, which supply the head. The fifth aortic arch is said not to usually develop in human embryos. The proximal part of the sixth aortic arch develops into the left pulmonary artery.

24 A 1-year-old child was admitted to the pediatric clinic due to severe dyspnea. ECG reveals cardiac arrhythmia and right ventricular hypertrophy. An angiogram reveals a patent ductus arteriosus (PDA). From which of the following embryologic arterial structures does the PDA take origin? A. Left sixth arch B. Right sixth arch C. Left fifth arch D. Right sixth arch E. Left fourth arch

24 A. The left sixth aortic arch is responsible for the development both of the pulmonary arteries and the ductus arteriosus. Without regression of the ductus arteriosus, a patent connection remains between aorta and the pulmonary trunk. The ductus arteriosus often reaches functional closure within 24 hours after birth, whereas anatomic closure and subsequent formation of the ligamentum arteriosum often occur by the twelfth postnataleek.

25 A 4-year-old girl is admit ed to the hospital with high fever. Staphylococcus aureus is isolated from her blood cultures and antibiotic therapy is initiated. A loud, harsh murmur is heard on auscultation. A chest radiograph shows prominent pulmonary arteries. Echocardiography shows all the valves to be normal. Which congenital heart disease most likely explains these findings? A. Atrial septal defect B. Tetralogy of Fallot C. Coarctation of the aorta D. Patent ductus arteriosus E. Aortic atresia

25 D. With a patent ductus arteriosus, an abnormal connection persists between the aorta and the pulmonary trunk. Blood leaving the left ventricle of the heart and into the aorta is reshunted back into the left pulmonary artery. This is responsible for the murmur heard during auscultation of the heart. The diversion of blood to the pulmonary arteries causes increased atrial pressure, leading to enlarged, and therefore noticeable, pulmonary arteries on the chest radiograph. The tetralogy of Fallot often presents with a right-to-left shunt of blood flow through the ventricles. It is also associated with pulmonary artery stenosis, right ventricular hypertrophy, interventricular septal defect, and an overriding of the aorta. This condition would not present with a murmur, however. Atrial septal defects are often characterized by a left-to-right shunt of blood, which often presents with dyspnea and abnormal heart sounds. A chest radiograph would not reveal prominent pulmonary arteries in such cases. Both aortic atresia and coarctation of the aorta result in a narrowing of the aorta but would not lead to noticeable prominent pulmonary arteries on the radiograph.

26 A 3-day-old infant is admitted to the cardiology unit with severe cyanosis. During echocardiographic examination a right-to-left shunt is identified. Which of the following conditions will most likely produce this type of shunt? A. Interatrial septal defect B. Interventricular septal defect C. Patent ductus arteriosus D. Corrected transposition of the great arteries E. Common truncus arteriosus

26 E. A common truncus arteriosus results from failure of separation of the pulmonary trunk and aorta. Without proper perfusion of the child by oxygenated blood, severe cyanosis will result.

27 A 4-day-old infant was admitted to the pulmonary unit suffering from dyspnea and cyanosis. Radiographic examination revealed a left hypoplastic lung and herniation of abdominal intestines into the left thoracic cavity. Which of the following embryologic structures most likely failed to develop properly? A. Septum transversum B. Pleuroperitoneal membrane C. Tracheoesophageal septum D. Laryngotracheal groove E. Foregut

27 B. The pleuroperitoneal membrane forms the posterolateral aspect of the diaphragm. A defect in this membrane would allow for communication between the upper left abdominal cavity and thoracic cavity and could result in a congenital diaphragmatic hernia. The septum transversum provides origin to the central tendon of the diaphragm but is not involved in herniation of the intestines. The tracheoesophageal septum, laryngotracheal groove, and oligohydramnios are not associated with development of the diaphragm.

28 A 3-month-old infant is diagnosed with a ventricular septal defect (VSD) at the area of the subpulmonary infundibulum. Which of the following structures must be avoided carefully by the surgeon when the sutures are placed at the site of the defect? A. Right bundle branch B. Right coronary artery C. Tricuspid valve D. Left anterior descending coronary artery E. Aortic valve

28 A. When closing a ventricular septal defect, it is important not to suture over the right bundle branch because it carries the stimulating impulse from the atrioventricular node to the apex of the heart through the right bundle of His. Following the course of the right bundle branch on the interventricular septum, the impulses travel along the septomarginal trabeculation (moderator band) and Purkinje fibers, leading to ventricular contraction. The right coronary artery passes dorsally in the atrioventricular groove; therefore, it does not pass through the interventricular septum. The anterior interventricular (left anterior descending) coronary artery is superficial to the IV septum on the anterior surface of the heart. The tricuspid valve and aortic valve are not directly associated with the interventricular septum.

29 A 2-day-old infant is diagnosed with incomplete division of the foregut into respiratory and digestive portions. Which is the most common congenital condition characteristic of this description? A. Esophageal atresia B. Esophageal achalasia C. Tracheoesophageal fistula D. Congenital diaphragmatic hernia E. Esophageal fistula

29 A. Esophageal atresia is often the result of an incomplete division of the tracheoesophageal septum, thus causing an absence of, or blind ending of, the esophagus. Though similar to an esophageal atresia, a tracheoesophageal fistula is an atypical connection between the trachea and the esophagus.

3 A 2-year-old child is seen in the pediatric cardiology unit for a congenital heart condition. Which of the following conditions occurs most often? A. Membranous ventricular septal defect B. Tetralogy of Fallot C. Muscular ventricular septal defect D. Ostium secundum defect E. Ostium primum defect

3 A. Ventricular septal defects account for 25% of congenital heart defects. The most common of these are defects in the membranous portion of the interventricular septum (membranous ventricular septal defects).

30 An unconscious 2-month-old infant is admitted to the emergency department after an automobile collision. An emergency tracheostomy is performed. Which of the following structures is most commonly at high risk of injury during this procedure? A. Left brachiocephalic vein B. Left common carotid artery C. Vagus nerve D. Phrenic nerve E. Thoracic duct

30 A. In a tracheotomy, an incision is made at the level of the sixth cervical vertebra, near the cricoid cartilage. The left brachiocephalic vein passes across the trachea immediately anterior to the brachiocephalic trunk. This vein is the most superficial structure and thus the most likely to be damaged. The left common carotid artery, the vagus nerve, and the phrenic nerve are not situated near the midline incision of the tracheotomy. The thoracic duct is located posterior and lateral to the esophagus and the trachea and is not likely to be damaged during a tracheotomy, other than the intentional opening made in it.

31 A 45-year-old female is admitted to the hospital with difficulty breathing. Radiographic examination reveals a tumor invading the lung surface anterior to the hilum. Which nerve is most likely compressed by the tumor to result in dyspnea? A. Phrenic B. Vagus C. Intercostal D. Recurrent laryngeal E. Cardiopulmonary

31 A. The phrenic nerve has a path between the anterior medial aspect of the lung and the mediastinum. Along the path of the nerve, it courses over the hilum of the lung. The vagus nerves run posterior to the heart as they give off branches to the cardiac plexus upon the trachea near the carina. The recurrent laryngeal nerves arise from the vagus nerves before the vagus nerves pass behind the hila of the lungs.

32 A 62-year-old male patient expresses concern that his voice has changed over the preceding months. Imaging reveals a growth located within the aortic arch, adjacent to the left pulmonary artery. Which neural structure is most likely being compressed to cause the changes in the patient's voice? A. Left phrenic nerve B. Esophageal plexus C. Left recurrent laryngeal nerve D. Left vagus nerve E. Left sympathetic trunk

32 C. The left recurrent laryngeal nerve passes beneath the ligamentum arteriosum and then loops superiorly toward the tracheoesophageal groove, medial to the arch of the aorta.

33 A 39-year-old woman visits the outpatient clinic and complains of inability to reach a pantry shelf just above her head. History reveals that 2 months ago she underwent a mastectomy procedure and she did not have this complaint prior to the surgery. Which nerve was most likely damaged during surgery to result in the patient's complaint? A. Axillary B. Spinal accessory C. Long thoracic D. Radial E. Thoracodorsal

33 C. During mastectomy procedures, three superficial nerves are susceptible to ligation or laceration: the longhoracic nerve, intercostobrachial nerve, and thoracodorsal nerve. In the event of injury to the long thoracic nerve, the patient complains of an inability to fully abduct the humerus above the horizontal. The serratus anterior (supplied by the long thoracic nerve) is necessary to elevate, rotate, and abduct the scapula, to facilitate abduction of the humerus above the shoulder. Because the patient does not indicate any loss of medial rotation or adduction of the humerus, ligation or injury of the thoracodorsal nerve can be eliminated.

34 A 41-year-old female is admitted to the emergency department with a complaint of severe, sharp, but poorly localized pain on the chest wall. Radiographic examination gives evidence of pleural effusion. What is the location of the neuronal cell bodies responsible for the nerve fibers that carry this pain to the central nervous system (CNS)? A. Dorsal root ganglia B. Sympathetic chain ganglia C. Dorsal horn of the spinal cord D. Lateral horn of the spinal cord E. Ventral horn of the spinal cord

34 A. The dorsal root ganglia contain nerve cell bodies for general somatic afferent and general visceral afferent neuronal processes. Pain localized on the chest wall is transmitted back to the CNS via sensory fibers.

35 A 23-year-old man is admitted to the emergency department after an automobile collision. Physical examination reveals tachycardia. What is the location of the preganglionic neural cell bodies involved in increasing the heart rate? A. Deep cardiac plexus B. Dorsal motor nucleus of vagus C. Lateral horn T5 to T9 D. Lateral horn T1 to T4 E. Superior, middle, and inferior cervical ganglia

35 D. The lateral horns, or intermediolateral cell columns, contain the cell bodies of preganglionic neurons of the sympathetic system. Spinal cord segments T1 to T4 are often associated with the upper limbs and thoracic organs; the autonomic neurons in spinal cord segments T5 to T9 usually correlate with innervation of organs in the abdominal cavity, specifically organs derived from the foregut.

36 A 55-year-old male is admitted to the emergency department with a diagnosis of possible myocardial infarction. Which nerves carry pain fibers from the heart to the CNS? A. Vagus B. Greater thoracic splanchnic C. Least thoracic splanchnic D. Cardiopulmonary (thoracic visceral) E. T5 to T9 ventral rami

36 D. The cardiopulmonary splanchnic (or thoracic visceral) nerves are responsible for carrying the cardiac sympatheticfferent fibers from the sympathetic ganglia to the thoracic viscera and afferent fibers for pain from these organs. The vagus nerve is responsible for carrying parasympathetic fibers. The greater and lesser splanchnic nerves carry sympathetic preganglionic fibers to the abdomen. T1 to T4 ventral rami receive sensory fibers forpain, carried initially by the cardiopulmonary nerves, en route to their respective final destination.

37 A 17-year-old girl is admitted to the hospital with severe dyspnea. Physical examination reveals that the patient is suffering from an asthma attack, with associated bronchospasm. Which of the following nerves is responsible for the innervation of the bronchial smooth muscle cells? A. Greater thoracic splanchnic B. Phrenic C. Vagus D. Intercostal E. Lesser thoracic splanchnic

37 C. The vagus nerve is the only nerve responsible for parasympathetic innervation of the lungs. The phrenic nerve and intercostal nerves are somatic nerves and are not involved in innervation of the heart or lungs. The greater thoracic splanchnic and lesser thoracic splanchnic nerves are responsible for carrying preganglionic sympathetic fibers for the innervation of the abdomen. They also carry afferents for pain from the abdomen.

38 A 42-year-old woman is admitted to the hospital with an inability to speak. The patient's personal history reveals that she has experienced hoarseness for the past month. A chest radiograph reveals a mass at the aortopulmonary window. Which of the following nerves is most likely compressed? A. Vagus B. Phrenic C. Left recurrent laryngeal D. Right recurrent laryngeal E. Greater thoracic splanchnic

38 C. There is close proximity between the aortopulmonary window and the left recurrent laryngeal nerve. A mass within or adjacent to this window is thus likely to compress the left recurrent laryngeal nerve, resulting in the hoarseness for the patient. The greater and lesser thoracic splanchnic nerves arise inferior and posterior to the aortopulmonary window and are thus unlikely to be compressed. The thoracic splanchnic nerves are not involved in the innervation of the larynx. Though the vagus is responsible for innervation of the larynx, it passes dorsal to the area of the aortopulmonary window and is not likely to be compressed.

39 Following the diagnosis of breast cancer, a 42-year-old woman underwent a total mastectomy, including excision of the axillary tail (of Spence). Postoperatively, the patient complains of dysesthesia in the inner aspect of the arm and axilla. Which of the following nerves was most likely injured during the procedure? A. Ulnar B. Long thoracic C. Intercostobrachial D. Lateral cutaneous nerve of T4 E. Axillary nerve

39 C. The intercostobrachial nerve is responsible for innervation of the skin on the medial surface of the arm. The ulnar nerve is responsible for cutaneous sensation on the medial aspect of the hand, and the axillary nerve innervates the lateral aspect of the shoulder. The lateral cutaneous branch of T4 innervates the dermatome corresponding to the nipple and areola and also supplies the medial aspect of the axilla. The long thoracic nerve provides motor supply to the serratus anterior and is not involved in cutaneous innervation of the axillary region. Only the intercostobrachial nerve is responsible for sensory supply of the lateral aspect of the axilla.

4 A 2-day-old is diagnosed with transposition of the great arteries. If this condition were to be left untreated for more than 4 months, it would be fatal. Which of the following structures must remain patent so that the infant can survive until surgical correction of the malformation? A. Ductus arteriosus B. Umbilical arteries C. Umbilical vein D. Coarctation of the aorta E. Pulmonary stenosis

4 A. In a case of transposition of the great arteries, oxygenated blood travels from the left ventricle into the pulmonary trunk, where it will eventually reach the lungs. In contrast, the aorta would be carrying deoxygenated blood into the systemic circulation. A patent ductus arteriosus acts as a shunt between the aorta and pulmonary trunk, allowing oxygenated and deoxygenated blood to mix and therefore allowing some oxygenated blood to reach the tissues. None of the other answer choices would correct this problem; with these structures remaining patent, the body would still not receive sufficient oxygenated blood for survival to be possible.

40 A 39-year-old male is admitted to the hospital with a complaint of severe retrosternal pain that radiates to the left shoulder. The pain is relieved by leaning forward. Auscultation reveals a pericardial friction rub, leading to a diagnosis of pericarditis. Which of the following nerves is responsible for the radiating pain? A. Intercostobrachial B. Phrenic C. Long thoracic D. Greater thoracic splanchnic E. Cardiopulmonary

40 B. Pericarditis is an inflammation of the pericardium and often causes a pericardial friction rub, with the surface of the pericardium becoming gradually coarser. Because the phrenic nerve is solely responsible for innervation of the pericardium, it would transmit the pain fibers radiating from the pericardial friction rub. The phrenic nerve contains sensory nerve fibers from C3 to C5, spinal nerve levels that also supply the skin of the shoulder area; therefore, pain carried by the phrenic nerve may be referred to the shoulder.

41 A 72-year-old male is admit ed to the hospital with complaints of severe chest pain radiating to his left arm. ECG examination provides evidence of significant myocardial infarction of the posterior wal of the left ventricle. Which of the fol owing nerves is responsible for the radiation of pain to the arm during myocardial infarction? A. Phrenic B. Vagus C. Intercostobrachial D. Greater splanchnic E. Suprascapular

41 C. The intercostobrachial nerve is the lateral cutaneous branch of the second intercostal nerve. It serves a sensory function both in the thoracic wall and medial aspect of the arm. The phrenic nerve arises from spinal nerves C3 to C5 and innervates the diaphragm. This nerve has no branches that pass into the arm. The vagus nerve is CN X and is a major supplier of autonomic function to the gut, up to the left colic flexure, and also provides some autonomic motor and sensory supply to organs in the head, neck, and thorax. The greater thoracic splanchnic nerve originates in the thorax from the sympathetic chain at the levels of T5 to T9 and innervates abdominal structures. The suprascapular nerve originates from the upper trunk of the brachial plexus and receives fibers primarily from C5 and C6. It innervates the supraspinatus and the infraspinatus.

42 A 43-year-old male hunter is admitted to the emergency department after falling over a barbed wire fence, as a result of which he suffered several deep lacerations along the left midaxillary line. When the patient is examined in the outpatient clinic several days later, numbness and anhydrosis are observed anterior to the area of the cuts. Which structures were most likely damaged to result in these signs? A. Dorsal roots B. Ventral roots C. Cutaneous branches of dorsal rami D. Cutaneous branches of ventral rami E. Rami communicans

42 D. Ventral rami contain both sensory and motor fibers and also sympathetics to the body wall, supplying all areas of the body wall except for tissues of the back. In this case both sensory fibers (numbness) and sympathetics (anhydrosis) are disrupted at the midaxillary line; therefore, cutaneous ventral rami is the only correct choice. The dorsal roots carry somatic and visceral sensory information from the periphery. Because only cutaneous sensation is lost the deficit cannot be the dorsal roots. The ventral roots of the spinal cord carry only somatic and visceral efferents. Because no motor functions are disrupted, this is not theorrect answer. The branches of dorsal rami provide cutaneous and postural muscle innervation to the back and thus have no relation to the midaxillary line. The rami communicans are components of the sympathetic nervous system and are not involved with general somatic afferent sensation.

43 A 62-year-old patient is admitted to the hospital with a complaint of suddenly occurring, tearing pain radiating to his back. A CT examination reveals that the patient has an aortic aneurysm. An urgent placement of an endovascular stent-graft is ordered. Which of the following nerves are most likely responsible for the tearing sensation radiating to his back? A. Somatic afferent B. Thoracic visceral afferent C. Sympathetic postganglionics D. Sympathetic preganglionics E. Parasympathetic afferent

43 D. General visceral afferents are nerve fibers that carry sensation from organs, in this case pain from the abdominal aorta. These fibers get mixed with general somatic afferents in the dorsal roots. This is the phenomenon of "referred pain." The dorsal root ganglia (or their counterparts associated with sensory cranial nerves) contain the cell bodies associ ated with all sensory fibers from the body, including somatic and visceral sensation.

44 A 22-year-old woman had undergone elective breast enhancement, with the insertion of 250-ml saline bags bilaterally. This resulted, unfortunately, in loss of sensation bilaterally in the nipples and areolae and some reduction of sensation of the skin from the areolae laterally to the midaxillary lines. Which of the following nerves were most likely subject to iatrogenic injury? A. Anterior cutaneous branches of second and third intercostal nerves B. Anterior and lateral cutaneous branches of the fourth intercostal nerves C. Lateral pectoral nerves D. Cutaneous branches of the second thoracic spinal nerves (intercostobrachial nerves) E. Lateral cutaneous branches of the second and third intercostal nerves

44 B. The anterior and lateral cutaneous branches of the fourth intercostal nerves provide the sensory and sympathetic supply to the areolae and nipples. Anterior cutaneous branches of the second and third intercostal nerves innervate the skin above the nipples and areolae. Lateral pectoral nerves provide motor innervation to the pectoralis major and minor, not sensory supply. Ventral primary rami of the second thoracic spinal nerves provide muscle innervation and sensory innervation above the nipples and areolae and sensory fibers for the medial side of the arm.

45 A 32-year-old female is admitted to the emergency department with dyspnea, dysphagia, hoarseness, and severe anxiety. Her medical history reveals that she has lived on a liquid diet for some months and has lost more than 30 lb. Over the past several weeks, she has had bloody sputum during attacks of coughing. Fluoroscopy and a barium swallow reveal a 4-cm mass associated with a bronchus and associated compression of the esophagus. Which of the following nerves is most likely to be affected? A. Right recurrent laryngeal nerve B. Left vagus nerve, posterior to the hilum of the lung C. Left recurrent laryngeal nerve D. Greater thoracic splanchnic nerve E. Phrenic nerve

45 C. The left recurrent laryngeal nerve passes superiorly in the tracheoesophageal groove after looping around the aorta. The compression of this nerve and compression of the esophagus against the trachea would result in the presenting symptoms. The right recurrent laryngeal nerve loops around the right subclavian artery before passing toward the larynx and therefore does not descend into the thorax. The left vagus nerve courses posterior to the hilum of the lung, after it has already given off its left recurrent laryngeal branch at the level of the aortic arch; therefore, compression of this nerve would not result in the presenting symptoms. The greater thoracic splanchnic nerve arises from sympathetic chain ganglia at levels T5 to T9 and therefore would not cause the presenting symptoms. The phrenic nerve innervates the diaphragm; compression of this nerve would not result in the presenting symptoms.

46 A 35-year-old man is admitted to the hospital with pain on swallowing. Imaging reveals a dilated left atrium. Which structure is most likely being compressed by the expansion of the left atrium to result in the patient's symptoms? A. Esophagus B. Root of the lung C. Trachea D. Superior vena cava E. Inferior vena cava

46 A. The patient's chief complaint is pain upon swallowing. With a dilated left atrium, the most probable structure being compressed is the esophagus. The esophagus descends into the abdomen immediately posterior to the left atrium below the level of the tracheal carina. The root of the lung is the site of junction at the hilum where the pulmonary arteries, veins, and bronchi enter or leave. The lung root is not so intimately associated with the esophagus and would not be associated with pain during swallowing. The trachea ends and bifurcates above the level of the left atrium and therefore would be unaffected by a dilated left atrium. The inferior vena cava ascends from the abdomen to the right atrium and the superior vena cava is quite anterior in position. Neither of these veins is closely related to the esophagus or the left atrium.

47 A 32-year-old female is admitted to the hospital in a comatose state. Physical examination reveals that the patient suffers from anorexia nervosa. A nasogastric tube is ordered to be inserted. What is the last site at which resistance would be expected as the tube passes from the nose to the stomach? A. Pharyngoesophageal junction B. Level of the superior thoracic aperture C. Posterior to the aortic arch D. Posterior to the left main bronchus E. Esophageal hiatus of the diaphragm

47 E. The esophageal hiatus in the diaphragm is one of four openings associated with the diaphragm. It is located at the level of T10 and allows the esophagus to pass through the thoracic cavity into the abdominal cavity. It is the most inferior of four esophageal constrictions. The pharyngoesophageal junction is the site at which the pharynx ends and the esophagus begins in the neck, at the level of the sixth cervical vertebra. It is the first and the most superior of the esophageal constrictions. There are no constrictions found at the level of the superior thoracic aperture; this is the opening for passage of the structures passing from the neck into the thorax. The esophagus descends posterior to the arch of the aorta. It is at this level that the second of the esophageal constrictions is found. The third constriction occurs as the esophagus passes posteriorly to the left main bronchus.

48 A 59-year-old man is admitted to the hospital with severe chest pain. During examination a slight rhythmic pulsation on the chest wall at the left fifth intercostal space is noted in the midclavicular line. What part of the heart is responsible for this pulsation? A. Right atrium B. Left atrium C. Aortic arch D. Apex of the heart E. Mitral valve

48 D. The apex of the heart is located in the left fifth intercostal space, about 3 1/2 inches to the left of the sternum. When this area of the heart is palpated, any pulsations would be generated by throbbing of the apex of the heart against the thoracic wall. This is also the location for performing auscultation (listening) of the mitral valve, not associated with palpation. The right atrium is located to the right of the sternum. The left atrium is located on the posterior aspect of the heart, thus no direct palpation is realized. The aortic arch would be located posterior to the manubrium of the sternum, above the second intercostal space.

49 A 42-year-old male was admitted to the hospital after a head-on vehicular collision in which he received severe blunt trauma to his sternum from the steering wheel. What part of the heart would be most likely to be injured by the impact? A. Right ventricle B. Apex of left ventricle C. Left ventricle D. Right atrium E. Anterior margin of the left atrium

49 A. These components of the heart are readily viewed in a plain radiograph of the thorax. It is important to understand the spatial arrangement of the heart as it rests in the thorax. The conus region of the right ventricle is located on the most anterior aspect of the heart, thus it is the most anterior portion of the heart within the thorax. The apex of the left ventricle is also located anteriorly, but it is located lateral to the sternum and occupies little area compared with the right ventricle. The left ventricle is positioned on the left lateral side and slightly posterior position in the thorax. The right atrium is located on the right lateral side of the heart. The anterior margin of the left atrium is positioned posteriorly in the thorax.

5 A 2-day-old newborn female is diagnosed with pulmonary stenosis, overriding of the aorta, ventricular septal defect, and hypertrophy of the right ventricle. Which condition is best characterized by these signs? A. Tetralogy of Fallot B. Atrial septal defect C. Transposition of the great vessels D. Pulmonary atresia E. Ventricular septal defect

5 A. Tetralogy of Fallot is always characterized by four cardiac defects: pulmonary stenosis, ventricular septal defect (VSD), overriding aorta, and these in turn lead to right ventricular hypertrophy. An atrial septal defect (ASD) is characterized by the communication between the two atria. In a case of transposition of the great vessels, the aorta arises from the right ventricle and the pulmonary trunk arises from the left ventricle.

50 A 54-year-old male is admit ed to the hospital with dyspnea. Imaging and physical examination and echocardiographic studies reveal severe mitral valve prolapse. Auscultation of this valve is best performed at which location? A. Left fifth intercostal space, just below the nipple B. Right lower part of the body of the sternum C. Right second intercostal space near the lateral border of the sternum D. Directly over the middle of the manubrium E. Left second intercostal space near the lateral border of the sternum

50 A. The left fifth intercostal space, just below the left nipple, is typically the location to listen to the mitral valve. Although the mitral valve is located at the fourth intercostalpace just to the left of the sternum, the sound is best realized "downstream" from the valve. The right lower part of the body of the sternum is the location of the tricuspid valve. The right second intercostal space near the lateral border of the sternum is the typical location of auscultationof the aortic valve. It is difficult to hear valvular sounds through bone, so auscultating directly over the middle of the manubrium is not a good choice. The left second intercostal space near the lateral border of the sternum is the site chosen typically for auscultation of the pulmonary valve.

51 A 48-year-old male patient is admitted with chronic angina. Coronary angiography reveals nearly total blockage of the circumflex artery near its origin from the left coronary artery. When this artery is exposed to perform a bypass procedure, what accompanying vein must be protected from injury? A. Middle cardiac B. Great cardiac C. Small cardiac D. Anterior cardiac E. Posterior cardiac

51 B. The great cardiac vein (anterior interventricular vein) takes a pathway initially beside the anterior interventricular coronary artery (left anterior descending: LAD) in its course, finally terminating in the coronary sinus when it is joined by the oblique vein of the left atrium (of Marshall). This vein must be protected when performing bypass procedures. The middle cardiac vein is located on the posterior aspect of the heart, but it also drains into the coronary sinus. The small cardiac vein drains blood along the same path as the right marginal branch. The anterior cardiac veins drain the blood from the right ventricle anteriorly and drain directly into the right atrium, and are not associated with the anterior interventricular artery.

52 A 55-year-old patient is to undergo a coronary bypass operation. The artery of primary concern is the vessel that supplies much of the left ventricle and the right and left bundle branches of the cardiac conduction system. Which artery is the surgeon most concerned with? A. Right marginal B. Anterior interventricular C. Circumflex D. Artery to the sinoatrial (SA) node E. Posterior interventricular

52 B. The anterior interventricular artery supplies the right and left ventricles and anterior two thirds of the IV septum. The right marginal artery supplies the right ventricle and apex of the heart; therefore, it does not supply the left ventricle. The left coronary circumflex artery supplies the left atrium and left ventricle; it courses posteriorly in, or near to, the coronary sulcus and supplies the posterior portion of the left ventricle and left atrium. The artery to the SA node is a branch of the right coronary artery and does not supply the left ventricle. The posterior interventricular (posterior descending) artery arises from the right coronary artery in 67% of people (this is referred to as a right dominant pattern) and supplies the posterior aspect of both ventricles and the posterior third of the interventricular septum.

53 A 58-year-old patient presents himself to the emergency department with severe angina. Upon cardiac catheterization, it is found that he has a significant occlusion in his right coronary artery, just distal to the right sinus of the aortic valve. His collateral cardiac circulation is minimal. Assuming the patient is right coronary dominant, which of the following arteries would be most likely to still have normal blood flow? A. Right (acute) marginal artery B. Atrioventricular nodal artery C. Posterior interventricular artery D. Sinoatrial nodal artery E. Anterior interventricular artery

53 E. The anterior interventricular artery arises from the left coronary artery. If there is occlusion in the right coronary artery, the anterior interventricular artery will still have normal blood flow. The right marginal artery branches from the right coronary artery; therefore, if there is occlusion of the right coronary artery, flow from the marginal artery will be compromised. The AV nodal artery is supplied by the coronary artery that crosses the crux of the heart posteriorly. If this artery arises from the right coronary, supply to the AV node might be reduced, depending upon collateral supply. The SA nodal artery is supplied by the right coronary artery in 55% of the population (only 35% from the left); inasmuch as it is stated that the patient is right coronary dominant, it would be predicted that the SAodal artery will not have normal blood flow.

54 A 55-year-old male is admitted to the emergency department with severe chest pain. Coronary angiography reveals that the patient's right coronary artery is free of pathology. The left coronary artery is found to be 70% to 80% occluded at three points proximal to its bifurcation into the circumflex and left anterior descending arteries. Having a left dominant coronary circulation, and without surgery, what is the most likely explanation for a poor prognosis for recovery of this patient to a normally active life? A. All the branches of the coronary artery are end arteries, precluding the chance that anastomotic connections will occur. B. It is probable that the anterior and posterior papillary muscles of the tricuspid valve have been damaged. C. The blood supply of the SA node is inadequate. D. The development of effective collateral circulation between anterior and posterior interventricular arteries will not be possible. E. The blood supply of the AV node will be inadequate.

54 D. Because the patient is left coronary artery dominant, if there is 70% to 80% occlusion of the left coronary, there will be deficiencies in flow both in the anterior descending and circumflex coronary arteries. No possibility is available for collateral flow from the posterior descending interventricular artery, for it too would be derived from the left coronary, by way of the circumflex artery. If the patient does not undergo surgery to remove or bypass the occlusion, he will be unable to have any substantial type of collateral circulation between the two major branches of the left coronary. The branches of the coronary arteries are not end arteries, and there are anastomoses between them. The papillary muscles of the tricuspid valve would not be affected with left coronary artery occlusion. The blood supply to the SA node would not be inadequate. The blood supply of the region of the AV node might or might not be adequate, for it could still be supplied by a branch of the right coronary artery.

55 A 35-year-old woman is admitted to the hospital with dyspnea. During physical examination her S1 heart sound is very loud. Which of the following valves is most likely defective? A. Mitral valve B. Aortic C. Pulmonary D. Aortic and pulmonary E. Tricuspid

55 A. The mitral valve corresponds to the S1 heart sound produced during systole. The aortic and pulmonary valves correspond to the S2 heart sound produced during diastole. The tricuspid valve also corresponds with the S1 heart sound. The aortic valve, however, corresponds with the S2 sound, so this answer would be incorrect.

56 A 72-year-old male is admitted to the hospital with severe chest pain. ECG examination provides evidence of severe myocardial infarction of the lower part of the muscular interventricular septum. The function of which of the following valves will be most severely affected? A. Pulmonary B. Aortic C. Tricuspid D. Mitral E. Eustachian

56 C. The interventricular septum is intimately involved with the tricuspid valve on the right side, via the muscular connections of the septomarginal trabeculum (moderator band) to the anterior papillary muscle. Therefore, if the electrical system of the heart is disrupted, as with a myocardial infarction in the upper portion of the muscular septum, the innervation of the interventricular septum will be compromised and the tricuspid valve will be directly affected. None of the other valves are directly involved with the interventricular septum.

57 A 35-year-old woman is admitted to the hospital with a complaint of shortness of breath. During physical examination it is noted that there is wide splitting in her S2 heart sound. ECG reveals a right bundle branch block. Which of the following valves is most likely defective? A. Mitral valve B. Pulmonary C. Aortic and mitral D. Tricuspid E. Tricuspid and aortic

57 B. The pulmonary valve is associated with the S2 heart sound produced in diastole. A splitting in the S2 sound indicates that the aortic and pulmonary valves are not closing simultaneously and would correlate with a possible defect in this valve. The mitral valve is associated with the S1 heart sound, produced in systole; therefore, it cannot be defective if only the S2 sound is involved. The aortic valve is associated with the S2 heart sound, but the mitral valve is not (as stated earlier); therefore, this answer cannot be correct. The tricuspid valve is associated with the S1 heart sound and therefore is not associated with the occurrence of an abnormal S2 heart sound.

58 A 3-month-old infant is diagnosed with a membranous ventricular septal defect. A cardiac operation is performed, and the septal defect is patched inferior to the noncoronary cusp of the aorta. Two days postoperatively the infant develops severe arrhythmias affecting both ventricles. Which part of the conduction tissue was most likely injured during the procedure? A. Right bundle branch B. Left bundle branch C. Bundle of His D. Posterior internodal pathway E. Atrioventricular node

58 C. The bundle of His is a collection of specialized cardiac muscle cells that carry electrical activity to the right and left bundle branches. Because both ventricles are affected, this is the logical site of injury, for this bundle leads to the bundle branches supplying both ventricles. An injury either to the right or left bundle branch would affect only one ventricle. Terminal Purkinje fibers transmit the electrical activity to the greater sections of the ventricles, yet dysfunction in the terminal part of the conduction system would affect only a small section of one ventricle, not both. The atrioventricular node is a group of specialized cardiac muscle cells that serve to decrease the rate of conduction to the ventricles and is located in the region deep to the septal wall of the right atrium. The posterior internodal pathway is in the roof of the right atrium and is not involved here.

59 A 62-year-old male was admitted to the hospital with intense left chest pain. ECG and echocardiography reveal myocardial infarction and pulmonary valve regurgitation. Emergency coronary angiography is performed and provides evidence that the artery supplying the upper portion of the anterior right ventricular free wall is occluded. Which of the following arteries is most likely to be occluded? A. Circumflex B. Anterior interventricular artery C. Posterior interventricular artery D. Artery of the conus E. Acute marginal branch of the right coronary artery

59 D. The artery of the conus is given off from the right coronary artery and winds around the conus arteriosus. The conus region is the superior part of the right ventricle that tapers into a cone (infundibulum) where the pulmonary valve leads into the pulmonary trunk. This conus artery supplies the upper portion of the anterior right ventricle and usually has a small anastomotic connection with the anterior interventricular (left anterior descending) branch of the left coronary artery. The circumflex artery supplies the left atrium and ventricle and does not supply the right ventricle except when the posterior interventricular (posterior descending) artery arises from the circumflex, or in unusual cases in which the circumflex passes to the surface of the right ventricle. The anterior interventricular artery supplies the right and left ventricles and the anterior two thirds of the IV septum. It is given off by the left coronary artery and does not specifically supply the upper portion of the right ventricle. The posterior interventricular artery supplies the right and left ventricles and the posterior third of the IV septum. It does not supply the upper portion of the right ventricle.

6 A 2-day-old newborn female is diagnosed with pulmonary stenosis, overriding of the aorta, ventricular septal defect, and hypertrophy of the right ventricle. Which of the following embryologic mechanisms is most likely responsible for the development of this cluster of anomalies? A. Superior malalignment of the subpulmonary infundibulum B. Defect in the aorticopulmonary septum C. Endocardial cushion defect D. Total anomalous pulmonary venous connections E. Atrioventricular canal malformation

6 A. Superior malalignment of the subpulmonary infundibulum causes stenosis of the pulmonary trunk. This leads to the four symptoms mentioned and is known as tetralogy of Fallot. A defect in formation of the aorticopulmonary septum is characteristic of transposition of the great arteries. An endocardial cushion defect is associated with membranous ventricular septal defects.

60 A 3-month-old male infant died unexpectedly in his sleep. The pathologist examined the histologic slides of tissue samples taken from the heart of the infant and observed that a portion of the conduction tissue that penetrates the right fibrous trigone had become necrotic. As a result, a fatal arrhythmia probably developed, leading to the death of the infant. Which of the following parts of the conduction tissue was most likely interrupted? A. Right bundle branch B. The bundle of Bachmann C. The left bundle branch D. The atrioventricular bundle of His E. The posterior internodal pathway

60 D. The atrioventricular bundle of His is a strand of specialized cardiac muscle fibers that arises from the atrioventricular node and passes through the right fibrous trigone. The right fibrous trigone (central fibrous body) is a dense area of connective tissue that interconnects the mitral, tricuspid, and aortic valve rings. After reaching the upper portion of the muscular interventricular septum, the bundle of His splits into right and left bundle branches. The bundle of Bachmann is a collection of fibers running from the sinoatrial node to the left atrium and is the only collection of conducting fibers to innervate the left atrium. Finally, the posterior internodal pathway, also known as Thorel's pathway, is the principal pathway of electrical activation between the sinoatrial node and atrioventricular node in humans.

61 A 42-year-old woman is admitted to the hospital after blunt trauma to her sternum by the steering wheel during a car crash. Radiographic examination reveals a cardiac tamponade. ECG data indicate that the heart has been severely injured. Which of the following cardiac structures will most likely be injured? A. Right ventricle B. Obtuse margin of the left ventricle C. Right atrium D. Left atrium E. Apex of the left ventricle

61 A. The sternocostal surface of the heart consists mostly of the right ventricle. Therefore, an anterior injury to the thorax would mostly likely first affect the right ventricle because it is adjacent to the deep surface of the sternum.

62 A 69-year-old male is admitted to the hospital with intense left chest pain. ECG reveals hypokinetic ventricular septal muscle, myocardial infarction in the anterior two thirds of the interventricular septum, and left anterior ventricular wall. The patient's ECG also exhibited left bundle branch block. Which of the following arteries is most likely occluded? A. Circumflex B. Proximal right coronary C. Proximal left coronary D. Proximal left anterior interventricular artery E. Posterior interventricular artery

62 D. The tissues affected in this case, the interventricular septum and anterior ventricular wall, are mostly supplied by the proximal portion of the left anterior interventricular artery. If the circumflex artery were blocked, the left atrium and left ventricle would be affected (in a right coronary dominant heart). If the right coronary artery were occluded, again assuming right coronary dominance, it would affect the right atrium, the sinoatrial and atrioventricular nodes, part of the posterior left ventricle, and the posterior part of the interventricular septum. If the left coronary artery (LCA) were blocked, most of the left atrium and left ventricle, the anterior two thirds of the interventricular septum, and the area of bifurcation of the bundle of His would be affected. If the posterior interventricular artery were occluded, it would affect the right and left ventricles and the posterior third of the interventricular septum. The circumflex and the anterior interventricular arteries are branches of the LCA, and the posterior interventricular artery is most commonly a branch of the terminal segment of the right coronary artery.

63 A 49-year-old woman is admit ed to the hospital complaining of severe, crushing, retrosternal pain during the preceding hour. An ECG reveals that she is suffering from acute myocardial infarction in the posterior aspect of her left ventricle and posteromedial papillary muscle. A coronary angiogram is performed and the patient is found to have left coronary dominant circulation. Which of the following arteries is the most likely to be occluded? A. Artery of the conus B. Right coronary artery C. Circumflex D. Right acute marginal E. Left diagonal

63 C. A "left coronary dominant" circulation means, most simply, that the left coronary artery (LCA) provides the posterior interventricular artery as a terminal branch of the coronary circumflex. The posterior aspect of the heart is composed primarily of the left ventricle and is supplied by the posterior interventricular branch. The artery of the conus supplies the right ventricular free wall. If the right coronary artery were occluded (in a right coronary dominant heart), it would affect the right atrium, right ventricle, the sinoatrial and atrioventricular nodes, the posterior part of the interventricular septum, and part of the posterior aspect of the left ventricle. The right acute marginal artery supplies the inferior margin of the right ventricle. The left diagonal arteries arise most commonly from the anterior interventricular (left anterior descending) artery but can also arise as branches of the left coronary or the circumflex.

64 A 75-year-old man is scheduled for his routine annual medical examination. During echocardiographic examination a large, mobile structure resembling a thrombus is identified in the right atrium near the opening of the inferior vena cava. After careful examination the doctor identifies the large mobile structure as a normal component of the heart. Which of the following structures could most likely resemble a thrombus in this location? A. Tricuspid valve B. Eustachian valve C. Thebesian valve D. Septum primum E. Fossa ovalis

64 B. The eustachian valve is an embryologic remnant of the valve of the inferior vena cava and is not a functional valve. The tricuspid valve is located below the inferior vena cava between the right atrium and right ventricle. The fossa ovalis is an embryonic remnant of the septum primum of the interatrial septum, located interatrially. The Thebesian valve is a semicircular fold at the orifice of the coronary sinus.

65 A 4-year-old male is operated on for a correction of a small, muscular interventricular septal defect. To access the right side of the intraventricular septum, a wide incision is first made in the anterior surface of the right atrium. Instruments are then inserted through the tricuspid valve to correct the ventricular septal defect. Which of the following structures is the most crucial to protect during the opening of the right atrium? A. Crista terminalis B. Pectinate muscles C. Tricuspid valve D. Eustachian valve E. Coronary sinus

65 A. The crista terminalis is a muscular ridge that runs from the opening of the superior vena cava to the inferior vena cava. This ridge provides the path taken by the posterior internodal pathway (of Thorel) between the sinoatrial and atrioventricular nodes. The crista also provides the origin of the pectinate muscles of the right auricle. The tricuspid valve is located below the inferior vena cava, between the right atrium and right ventricle. The eustachian valve is an embryologic remnant of the valve of the inferior vena cava. The ostium of the coronary sinus is located between the right atrioventricular orifice and the inferior vena cava.

66 A 52-year-old patient is admitted to the hospital with severe chest pain. ECG and radiographic examinations provide evidence of a significant myocardial infarction and cardiac tamponade. An emergency pericardiocentesis is ordered. At which of the following locations will the needle best be inserted to relieve the tamponade? A. Right seventh intercostal space in the midaxillary line B. Left fifth intercostal space in the midclavicular line C. Right third intercostal space, 1 inch lateral to the sternum D. Left sixth intercostal space, 1/2 inch lateral to the sternum E. Triangle of auscultation

66 B. During pericardiocentesis, the needle is inserted below the xiphoid process, or in the left fifth intercostal space in the midclavicular line. The most effective way of draining the pericardium is by penetrating the thoracic wall at its lowest point anatomically, hence the third intercostal space would be too cranial in position. The sixth and seventh intercostal spaces are locations that are not used clinically because of the increased likelihood of injury to the pleura or lungs and other complications.

67 A 55-year-old man is brought to the emergency department after his motorcycle collided with an automobile. He is hypotensive, his pulse is irregular, and he shows other signs of substantial blood loss. MRI and CT scan evaluations reveal profuse abdominal bleeding. A decision is made to enter the chest so that the descending thoracic aorta can be clamped to minimize blood loss and to preserve cerebral blood flow. After surgical entrance into the thorax, the fibrous pericardium is elevated with a forceps and punctured. A midline, longitudinal incision of the pericardium would best be made to prevent injury to which of the following structures? A. Auricular appendage of the left atrium B. Coronary sinus C. Left anterior descending artery D. Left phrenic nerve E. Left sympathetic trunk

67 C. The anterior interventricular (left anterior descending) artery lies anteriorly and to the left and descends vertically to the left toward the apex. It can be more easily injured by a transverse incision of the pericardium, which would cross perpendicular to this artery. The auricular appendage of the left atrium is located posteriorly; therefore, it would not be injured in an anterior longitudinal incision. The coronary sinus is between the right atrioventricular orifice and the inferior vena cava and would not be affected. The left phrenic nerve lies between the heart and the left lung and is too deep to be injured in this incision. The left sympathetic trunk is also too posterior to be injured.

68 During cardiac surgery of a 45-year-old male the cardiac surgeon can place her fingers in the transverse pericardial sinus, if necessary. This allows the surgeon to easily place a vascular clamp upon which of the following vessels? A. Right and left pulmonary veins B. Superior and inferior vena cava C. Right and left coronary arteries D. Pulmonary trunk and ascending aorta E. Pulmonary trunk and superior vena cava

68 D. A finger passing through the transverse pericardial sinus passes directly behind the great arteries exiting the heart, allowing the surgeon to rather easily place a vascular clamp upon the pulmonary trunk and ascending aorta. The other vessels listed are not readily accessible by way of the transverse sinus.

69 A 48-year-old male patient is scheduled to have a coronary arterial bypass because of chronic angina. Coronary arteriography reveals nearly total blockage of the posterior descending interventricular artery. In exposing this artery to perform the bypass procedure, which accompanying vessel is most susceptible to injury? A. Middle cardiac vein B. Great cardiac vein C. Small cardiac vein D. Anterior cardiac vein E. Coronary sinus

69 A. The middle cardiac veins run parallel with the posterior interventricular (posterior descending) artery and drains directly into the coronary sinus. The great cardiac vein parallels the anterior interventricular artery and the small cardiac veins pass parallel with the right marginal artery. The anterior cardiac veins are several small veins that drain directly into the right atrium. The coronary sinus is a wide venous channel that runs from left to right in the posterior part of the coronary groove.

7 A 5-year-old boy is admitted to the hospital with severe dyspnea. During physical examination a loud systolic murmur and a wide, fixed, split S2 sound is noted. What is the most likely diagnosis? A. Ventricular septal defect B. Atrial septal defect C. Tetralogy of Fallot D. Transposition of the great arteries E. Aortic stenosis

7 B. The murmur at S2 localizes the defect at an atrioventricular valve. An atrial septal defect causes a diastolic murmur in the tricuspid valve, whereas a ventricular septal defect would cause a pansystolic murmur. Transposition of the great arteries and aortic stenosis will cause a murmur at S1, and tetralogy of Fallot does not cause a murmur at S1 or S2 .

70 A 54-year-old male is admitted to the hospital with severe chest pain. ECG examination reveals a myocardial infarction. If the posterior interventricular branch in the patient arises from the right coronary artery, which part of the myocardium will most likely have its blood supply reduced if the circumflex branch of the left coronary artery becomes occluded from an atherosclerotic plaque? A. Anterior part of the interventricular septum B. Diaphragmatic surface of the right ventricle C. Infundibulum D. Lateral wall of the left ventricle E. Posterior part of the interventricular septum

70 D. The left coronary artery bifurcates into the anterior interventricular artery (left anterior descending: LAD) and the coronary circumflex branch. The circumflex branch gives off the left marginal branch, which supplies the lateral wall (obtuse margin) of the left ventricle. The anterior part of the interventricular septum is supplied by the LAD. The diaphragmatic surface of the right ventricle is supplied by the posterior descending artery and the right marginal, a branch of the right coronary artery. The infundibulum, also known as the conus arteriosus, is the outflow portion of the right ventricle. The posterior part of the interventricular septum is supplied by the posterior descending artery, in most cases a branch of the right coronary artery.

71 A 70-year-old male with a history of two previous myocardial infarctions is admitted to the hospital with severe chest pain. ECG reveals a new myocardial infarction and ventricular arrhythmia. Coronary angiography reveals that the right coronary artery is blocked just distal to the origin of the right marginal artery in a right coronary dominant circulation. Which of the following structures would most likely be affected after such a blockade? A. Right atrium B. SA node C. AV node D. Lateral wall of the left ventricle E. Anterior interventricular septum

71 C. The atrioventricular (AV) node is most commonly supplied by a branch of the right coronary artery. This branch arises at the crux of the heart (the point of junction of all four cardiac chambers posteriorly); this is the location of the occlusion. The right atrium is supplied by the right coronary artery, which additionally supplies the sinoatrial node. The left marginal artery supplies the lateral wall of the left ventricle. The anterior portion of the interventricular septum is supplied by the anterior interventricular artery.

72 A 43-year-old woman is diagnosed with mitral valve stenosis. During physical examination the first heart sound is abnormally loud. Which of the following heart valves are responsible for the production of the first heart sound? A. Aortic and mitral B. Aortic and tricuspid C. Tricuspid and mitral D. Mitral and pulmonary E. Tricuspid and pulmonary

72 C. The first heart sound is caused by the closure of the tricuspid and mitral valves. The second heart sound is caused by the closure of the aortic and pulmonary valves.

73 A 75-year-old woman is admitted to the hospital with anginal pain. ECG reveals myocardial infarction and a right bundle branch block. During physical examination the patient has a loud second heart sound. Which of the following heart valves are responsible for the production of the second heart sound? A. Aortic and pulmonary B. Aortic and tricuspid C. Tricuspid and mitral D. Mitral and pulmonary E. Tricuspid and pulmonary

73 A. The second heart sound is caused by the closure of the aortic and pulmonary valves. The first sound by the heart is caused by the closure of the tricuspid and mitral valves.

74 Ten days after a surgical procedure to correct her cardiac malformation, a 3-month-old infant died unexpectedly in her sleep. After an autopsy, the pathologist reported as follows: "A significant portion of the conduction tissue was found to be necrotic. The area of the necrotic tissue was located inferior to the central fibrous body, membranous septum, and septal leaflet of the tricuspid valve. Further examination revealed infarction of the surrounding tissue. The rest of the heart was unremarkable." Which of the following arteries was most likely occluded? A. Artery of the conus B. SA node artery C. AV node artery D. First septal perforator of the anterior interventricular artery E. All of the above

74 D. The first septal perforating branch of the anterior interventricular artery (left anterior descending: LAD) is the first branch of the LAD that supplies the conducting tissue of the heart; it passes directly to the point of bifurcation of the common atrioventricular bundle of His. The other vessels listed have no anatomic relation to the area of ischemia.

75 A 55-year-old male is undergoing an aortic valve replacement. During the procedure the heart is connected to the heart lung machine. As the surgeon explores the oblique pericardial sinus, which of the following is not directly palpable with the tips of the fingers? A. Inferior vena cava B. Superior vena cava C. Posterior wall of the left atrium D. Inferior right pulmonary vein E. Right atrium

75 B. The superior vena cava empties into the right atrium on the superior aspect of the heart; it is not directly palpable from the oblique sinus. The oblique sinus is a blind cul-de-sac providing access to the inferior vena cava, the posterior wall of the left atrium, right atrium, and the right and left pulmonary veins.

76 A 42-year-old female is admitted urgently to the emergency department after suffering a penetrating wound to her chest from an ice pick during a violent domestic dispute. Physical and ultrasound examinations reveal that the patient has cardiac tamponade. Which of the following will most likely be found during physical examination? A. There will be a visible or palpable decrease in the dimensions of the external jugular and internal jugular vein. B. There will be gradual enlargement of the ventricles in diastole. C. The difference between systolic and diastolic arterial pressures will increase significantly. D. There will be diminished heart sounds. E. The pulses in the internal carotid arteries will become increasingly distinct, as detected behind the angles of the mandible.

76 D. Cardiac tamponade is characterized by hypotension, tachycardia, muffled heart sounds, and jugular vein distention. Bleeding into the pericardial cavity would muffle the heart sounds because of the increased distance between the chest wall and the heart, leading to "distant" heart sounds. When the effusion is particularly severe, the heart may take on a "water bottle" appearance on an anterior-posterior radiograph.

77 During surgical repair of a congenital cardiac anomaly in a 15-year-old boy with a right dominant coronary arterial system, the surgeon accidentally injured a vessel that usually supplies part of the conduction system. This results in intermittent periods of atrioventricular block and severe arrhythmia. The injured artery was most likely a direct branch of which of the following arteries? A. Distal anterior interventricular artery B. Circumflex artery C. Left coronary artery D. Marginal artery E. Right coronary artery

77 E. "Right coronary dominant circulation" refers simply to the fact that the right coronary artery provides origin for the posterior interventricular (posterior descending) coronary artery. In such cases, it provides supply for the sinoatrial and atrioventricular nodes. It might be anticipated that right coronary blockage could result in dysfunction of the atrioventricular node, if collateral supply is poor or absent. The LAD, circumflex, and left marginal are all branches of the left coronary artery. The right marginal artery marginal is a branch of the right coronary artery.

78 A 42-year-old female is admitted to the hospital with dyspnea. Imaging reveals severe mitral valve regurgitation. Which of the following structures prevents regurgitation of the mitral valve cusps into the left atrium during systole? A. Crista terminalis B. Crista supraventricularis C. Pectinate muscles D. Chordae tendineae E. Trabeculae carneae

78 D. The chordae tendineae are fibrous cords that connect papillary muscles to valve leaflets. The restraint provided by these cords on the valve leaflets prevents the prolapse of the mitral valve cusps into the left atrium. The crista terminalis is a ridge that runs from the opening of the inferior vena cava to the superior vena cava. Trabeculae carneae are irregular ridges of myocardium that are present within the ventricles.

79 A 58-year-old female with cardiac arrhythmia has undergone a procedure to implant a pacemaker. The electrical conducting leads for the pacemaker must be passed into the heart from the pacemaker. Which of the following is the correct order of structures for passage of the leads into the right ventricle? A. Brachiocephalic vein, superior vena cava, mitral valve, right ventricle B. Superior vena cava, right atrium, mitral valve, right ventricle C. Superior vena cava, right atrium, tricuspid valve, right ventricle D. Brachiocephalic vein, superior vena cava, right atrium, tricuspid valve, right ventricle E. Brachiocephalic vein, superior vena cava, right atrium, mitral valve, right ventricle

79 D. The correct path that leads to the right ventricle for the lead of the pacemaker is the brachiocephalic vein (could be right or left; pacemakers are more commonly placed on the left so would be left brachiocephalic vein), superior vena cava, right atrium, tricuspid valve, and right ventricle.

8 A 3-month-old infant is diagnosed with Down syndrome. A routine cardiovascular examination reveals that the infant suffers from arrhythmias. What other cardiac conditions are most likely to occur with Down syndrome? A. Tetralogy of Fallot B. Transposition of the great arteries C. Atrial septal and ventricular septal defects D. Truncus arteriosus E. Coarctation of the aorta

8 C. Down syndrome (more properly called "trisomy 21") is associated with cardiovascular abnormalities such as arrhythmias and atrial and ventricular septal defects. It is also characterized by mental retardation, brachycephaly, flat nasal bridge, upward slant of the palpebral fissure, protruding tongue, simian crease, and clinodactyly of the fifth digit.

80 A 68-year-old male patient in the cardiology ward complains at each mealtime of difficulty in swallowing (dysphagia). Radiographic studies reveal significant cardiac hypertrophy. A barium swallow, followed by radiographic examination of the thorax, reveals esophageal constriction directly posterior to the heart. Which of the following is the most likely cause of the patient's dysphagia? A. Mitral valve stenosis B. Pulmonary valve stenosis C. Regurgitation of the aorta D. Occlusion of the anterior interventricular artery E. Occlusion of the posterior interventricular artery

80 A. Mitral stenosis leads to left atrial dilation, which can exert a compressive effect on the esophagus. The pulmonary valve is located between the outflow tract of the right ventricle and the pulmonary trunk. The aortic valve is located between the left ventricle and the aorta. Anterior interventricular (left anterior descending) and posterior interventricular (posterior descending) arterial occlusions can cause a myocardial infarction, but not dysphagia. In the normal position of the heart the left atrium lies most posteriorly. Therefore, a stenosis of the mitral valve (atrioventricular valve between left atrium and left ventricle) would lead to enlargement of the left atrium, which would in turn impinge upon the esophagus. A stenosis of the pulmonary valve would have no effect upon the esophagus because of the anterior position of the pulmonary trunk in the thorax. Regurgitation through any valve will ultimately decrease systemic blood flow. An occlusion of a coronary artery will lead to ischemia and possibly myocardial infarction.

81 A 35-year-old female is admitted to the emergency department because of cardiac arrhythmia. ECG examination reveals that the patient suffers from atrial fibrillation. Where is the mass of specialized conducting tissue that initiates the cardiac cycle located? A. At the junction of the coronary sinus and the right atrium B. At the junction of the inferior vena cava and the right atrium C. At the junction of the superior vena cava and the right atrium D. Between the left and right atria E. In the interventricular septum

81 C. The SA node, the primary pacemaker of the heart, is a mass of specialized cardiac cells within the myocardium at the upper end of the crista terminalis, near the opening of the superior vena cava into the right atrium. The AV node is at the junction of the coronary sinus and the right atrium upon the right fibrous trigone (central fibrous body). The eustachian valve directs blood from the inferior vena cava and through the right atrium toward the tricuspid valve ostium. The interatrial septum is located between the left and right atria. The septomarginal trabeculum (moderator band) arises from the muscular portion of the interventricular septum and passes to the base of the anterior papillary muscle in the right ventricle. The moderator band carries the right bundle branch of the conduction system just beneath its endocardial layer.

82 A 45-year-old female is admitted to the hospital with swelling (edema) of the lower limbs. Ultrasound examination reveals an incompetent tricuspid valve. Into which area will regurgitation of blood occur in this patient? A. Pulmonary trunk B. Left atrium C. Ascending aorta D. Right atrium E. Left ventricle

82 D. The tricuspid valve is the atrioventricular valve located between the right atrium and right ventricle. An incompetent valve would allow blood to regurgitate into the right atrium during systole and subsequently raise pressure in the venous system, increasing capillary pressure and causing edema. A regurgitation of blood into the pulmonary trunk would be a result of an incompetent pulmonary valve. Regurgitation of blood from the left ventricle back into the left atrium is a result of prolapse of the mitral valve. There is no direct anatomic relationship between the tricuspid valve and the ascending aorta. Blood would pool in the left ventricle in the event of aortic valve incompetence.

83 A 34-year-old male with a complaint of sharp, localized pain over the thoracic wall is diagnosed with pleural effusion. Through which intercostal space along the midaxillary line is it most appropriate to insert a chest tube to drain the effusion fluid? A. Fourth B. Sixth C. Eighth D. Tenth E. Twelfth

83 C. To avoid damaging the lungs, a chest tube should be placed below the level of the lungs, in the costodiaphragmatic recess. Such a point of entrance for the tube would be the eighth or ninth intercostal space. At the midclavicular line, the costodiaphragmatic recess is localized between intercostal spaces 6 and 8, at the midaxillary line between 8 and 10, and at the paravertebral line between ribs 10 and 12.

84 A 51-year-old male is admitted to the hospital with severe dyspnea. Radiographic examination reveals a tension pneumothorax. Adequate local anesthesia of the chest wall prior to insertion of a chest tube is necessary for pain control. Of the following layers, which is the deepest that must be infiltrated with the local anesthetic to achieve adequate anesthesia? A. Endothoracic fascia B. Intercostal muscles C. Parietal pleura D. Subcutaneous fat E. Visceral pleura

84 C. The parietal pleura is innervated by the intercostal nerves and is very sensitive to pain, in this case being somatic innervation. Therefore, the parietal pleura is the deepest layer that must be anesthetized to reduce pain during aspiration or chest tube placement.

85 A 5-year-old boy had been playing with his little race cars. Soon after he put a wheel from one of the cars in his mouth, he began choking and coughing. Where in the tracheobronchial tree is the most common site for a foreign object to lodge? A. The right primary bronchus B. The left primary bronchus C. The carina of the trachea D. The beginning of the trachea E. The left tertiary bronchus

85 A. The right main bronchus is the shorter, wider, and more vertical primary bronchus. Therefore, this is most often the location that foreign objects will likely be lodged. The left primary bronchus is not as vertical and therefore does not present the path of least resistance. (It must beunderstood, however, that in some cases of aspiration, the foreign body can pass into the left primary bronchus rather than the right bronchus!) The carina is a ridge separating the openings of left and right bronchi, the "fork in the road," so to speak. The trachea is a tubular structure supported by incomplete cartilaginous rings, and the likelihood that an object will be lodged there is minimal. It is unlikely that a foreign object would descend so far as to obstruct a tertiary bronchus, although this could happen.

86 A 3-year-old child is admitted to the emergency department with a particularly severe attack of asthma. Which of the following is the most important factor in increasing the intrathoracic capacity in inspiration? A. "Pump handle movement" of the ribs—thereby increasing anterior-posterior dimensions of the thorax B. "Bucket handle movement" of the ribs— increasing the transverse diameter of the thorax C. Straightening of the forward curvature of the thoracic spine, thereby increasing the vertical dimensions of the thoracic cavity D. Descent of the diaphragm, with protrusion of the abdominal wall, thereby increasing vertical dimensions of the thoracic cavity E. Orientation and flexibility of the ribs in the baby, thus allowing expansion in all direction

86 D. Contraction of the diaphragm (descent) pulls the dome inferiorly, increasing the vertical dimension of the thorax. This is the most important factor in inspiration for increasing the internal pulmonary volume and concomitantly decreasing intrathoracic pressure. The contraction of intercostal muscles is usually involved in forced inspiration, resulting in increases in the transverse and anteroposterior dimensions of the thoracic cavity.

87 A 54-year-old female is admitted to the hospital with a stab wound of the thoracic wall in the area of the right fourth costal cartilage. Which of the following pulmonary structures is present at this site? A. The horizontal fissure of the left lung B. The horizontal fissure of the right lung C. The oblique fissure of the left lung D. The apex of the right lung E. The root of the left lung

87 B. The horizontal fissure of the right lung is a fissure separating the superior lobe from the middle lobe. It usually extends medially from the oblique fissure at the midaxillary line to the sternum, along the lower border of the fourth rib. The apex of the right lung reaches to a level above the clavicle and is therefore superior to the stab wound in the fourth costal cartilage. The other answers are related to features of the left lung, which is not addressed in the question.

88 A 55-year-old female visited her doctor because of a painful lump in her right breast and a bloody discharge from her right nipple. Radiographic studies and physical examination reveal unilateral inversion of the nipple, and a tumor in the right upper quadrant of the breast is suspected. In addition, there is an orange-peel appearance of the skin (peau d'orange) in the vicinity of the areola. Which of the following best explains the inversion of her nipple? A. Retention of the fetal and infantile state of the nipple B. Intraductal cancerous tumor C. Retraction of the suspensory ligaments of the breast by cancer D. Obstruction of the cutaneous lymphatics, with edema of the skin E. Inflammation of the epithelial lining of the nipple and underlying hypodermis

88 C. The patient's symptoms are all indicative of inflammatory breast cancer. Common symptoms include inversion of the nipple and dimpling of the overlying skin, changes that are due to the retraction of the suspensory ligaments (of Cooper). Intraductal cancerous tumors show symptoms including breast enlargement, breast lump, breast pain, and nipple discharge. The other answers are not cancerous conditions.

89 A 58-year-old woman is admitted to the emergency department with severe dyspnea. Bronchoscopy reveals that the carina is distorted and widened. Enlargement of which group of lymph nodes is most likely responsible for altering the carina? A. Pulmonary B. Bronchopulmonary C. Inferior tracheobronchial D. Superior tracheobronchial E. Paratracheal

89 C. The inferior tracheobronchial nodes are also known as the carinal nodes and are located on the inferior aspect of the carina, the site of bifurcation of the trachea. The pulmonary nodes lie on secondary bronchi. The bronchopulmonary (hilar) nodes run along the primary bronchi. The superior tracheobronchial nodes are at the junction of the bronchi and the trachea. The paratracheal nodes run along the trachea.

9 A 3-month-old infant is diagnosed with a deletion at the 22q11 chromosome. A routine cardiovascular examination reveals severe congenital cardiac malformation. Which of the following malformations will most likely be associated with 22q11 syndrome? A. Tetralogy of Fallot and truncus arteriosus B. Transposition of the great arteries C. Atrial septal and ventricular septal defects D. Coarctation of the aorta E. Aortic atresia

9 A. Tetralogy of Fallot and truncus arteriosus are associated with DiGeorge syndrome (22q11). Transposition of the great arteries is associated with maternal diabetes. ASDs and VSDs are present in individuals with Down syndrome. Coarctation of the aorta is related to Turner syndrome. Marfan syndrome is present in individuals with aortic atresia.

90 A 72-year-old patient vomited and then aspirated some of the vomitus while under anesthesia. On bronchoscopic examination, partially digested food is observed blocking the origin of the right superior lobar bronchus. Which of the following groups of bronchopulmonary segments will be affected by this obstruction? A. Superior, medial, lateral, medial basal B. Apical, anterior, posterior C. Posterior, anterior, superior, lateral D. Apical, lateral, medial, lateral basal E. Anterior, superior, medial, lateral

90 B. The superior lobar bronchus is one of the divisions of the right main bronchus. This bronchus branches into apical, anterior, and posterior tertiary bronchi.

91 A 35-year-old woman is admitted to a surgical ward with a palpable mass in her right breast and swollen lymph nodes in the axilla. Radiographic studies and biopsy reveal carcinoma of the breast. Which group of axillary lymph nodes is the first to receive lymph drainage from the secretory tissue of the breast and therefore most likely to contain metastasized tumor cells? A. Lateral B. Central C. Apical D. Anterior (pectoral) E. Posterior (subscapular)

91 D. Lymphatic drainage of the breast is typically to the axillary nodes, more specifically to the anterior (pectoral) nodes. Lymphatic vessels from the pectoral nodes continue into the central axillary nodes, the drainage of which passes farther into the apical node, just inferior to the clavicle in the deltopectoral triangle. From these nodes lymph passes to the "sentinel," or scalene, nodes and the subclavian lymph trunk. The lateral and posterior axillary nodes do not normally receive lymph drainage from the breast but do receive lymph from the upper limb. (This is the reason for the edema of the upper limb that occurs after a mastectomy, in which there may be a total removal of axillary lymph nodes.)

92 A 30-year-old man is admitted to the emergency department because of significant nosebleeding and a headache that has worsened over several days. He also complains of fatigue. Upon examination it is noted that brachial artery pressure is markedly increased, femoral pressure is decreased, and the femoral pulses are delayed. The patient shows no external signs of inflammation. Which of the following is the most likely diagnosis? A. Coarctation of the aorta B. Cor pulmonale C. Dissecting aneurysm of the right common iliac artery D. Obstruction of the superior vena cava E. Pulmonary embolism

92 A. Increased arterial pressure in the upper limbs (as demonstrated in the brachial artery) and decreased pressure in the lower limbs (as demonstrated in the femoral artery) are common symptoms of coarctation of the aorta. Other symptoms include tortuous and enlarged blood vessels above the coarctation and an increased risk of cerebral hemorrhage. This condition of coarctation occurs when the aorta is abnormally constricted during development. The patient does not complain of respiratory distress, so cor pulmonale would not likely be the underlying condition. Dissection of the right common iliac artery would not result in nosebleed or headache. Obstruction of the superior vena cava would not account for decreased femoral pulse. A pulmonary embolism will not present with these findings.

93 A 22-year-old man is diagnosed with signs of reduced aortic flow. Upon examination it is noted that brachial artery pressure is markedly increased, femoral pressure is decreased, and the femoral pulses are delayed. The patient shows no external signs of inflammation. Which of the following conditions will most likely be observed in a radiographic examination? A. Flail chest B. Pneumothorax C. Hydrothorax D. Notching of the ribs E. Mediastinal shift

93 D. The diagnosis for these symptoms is coarctation of the aorta. This condition occurs when the aorta is abnormally constricted. One of the cardinal signs is a characteristic rib notching. "Notching" of the ribs is due to the reversal of direction of blood flow through the anterior intercostal branches of the internal thoracic artery, as these usually small arteries carry collateral arterial blood flow to the lower thoracic portion of the aorta inferior to the coarctation. Enlargement and vibration of the intercostal arteries against the rib results in erosion ("notching") of the subcostal grooves, which is visible on radiography.

95 A 22-year-old woman sustained a chest injury upon impact with the steering wheel during a car crash. Upon admission of the patient to the hospital, physical examination revealed profuse swelling, inflammation, and deformation of the chest wall. A radiograph revealed an uncommon fracture of the manubrium at the sternomanubrial joint. Which of the following ribs would be most likely to also be involved in such an injury? A. First B. Second C. Third D. Fourth E. Fifth

95 B. The superior margin of the manubrium is characterized by the jugular notch. Laterally are the sternoclavicular joints and the articulations of the first ribs with the manubrium. The second pair of ribs articulates with the sternum at the sternal angle, the junction of the manubrium with the body of the sternum. GAS 144-147; GA 58-61

96 A 47-year-old male is admitted to the emergency department, due to severe dysphagia. Edema of the lower limbs is apparent upon physical examination. A barium sulfate swallow imaging procedure reveals esophageal dilation, with severe inflammation, due to constriction at the esophageal hiatus. What is the most likely cause of the severe edema of the lower limbs? A. Thoracic aorta constriction B. Thoracic duct blockage C. Superior vena caval occlusion D. Aortic aneurysm E. Femoral artery disease

96 B. The thoracic duct is important in lymph drainage of the entire body with the exception of the upper right quadrant. The thoracic duct ascends between the aorta and azygos vein behind the esophagus. Dilation of the esophagus here in the lower thorax can compress the thoracic duct, leading to impairment of lymphatic drainage and resultant edema. GAS 219-220; GA 71

97 In coronary bypass graft surgery of a 49-year-old female, the internal thoracic artery is used as the coronary artery bypass graft. The anterior intercostal arteries in intercostal spaces three to six are ligated. Which of the following arteries will be expected to supply these intercostal spaces? A. Musculophrenic B. Superior epigastric C. Posterior intercostal D. Lateral thoracic E. Thoracodorsal

97 C. The anterior intercostal arteries anastomose with the posterior intercostal arteries. Ligation of the anterior arteries would not affect the supply of the intercostal spaces because the posterior arteries would provide collateral arterial supply. Branches of the musculophrenic artery provide supply for the lower seventh, eighth, and ninth intercostal spaces. The superior epigastric artery passes into the rectus sheath of the anterior abdominal wall. The lateral thoracic artery arises from the second part of the axillary artery, and the thoracodorsal artery is a branch of the subscapular artery, a branch of the third part of the axillary artery.

98 A 10-year-old boy is admitted to the hospital with retrosternal discomfort. A CT scan reveals a midline tumor of the thymus gland. Which of the following veins would most likely be compressed by the tumor? A. Right internal jugular B. Left internal jugular C. Right brachiocephalic D. Left brachiocephalic E. Right subclavian

98 D. The thymus lies in the superior mediastinum and extends upward into the neck, especially in the young. A midline tumor of this gland can compress the left brachiocephalic vein. The subclavian vein is distal or lateral to this location, and the thymus gland would not likely impinge upon it. The internal jugular veins are located superior and lateral to the position of the thymus gland. A midline tumor is more likely to cause compression of the left brachiocephalic vein, which crosses the midline, than the right brachiocephalic vein, which is not located in the midline.


Ensembles d'études connexes

Unit 8. Exam Investment Considerations for Small-Business Owners

View Set

Training Session 10 (Chapter 10)

View Set

EVRN 148 ch 12 nonrenewable energy questions

View Set

Ball State University FIN100 Final Exam

View Set